WBR0943

Revision as of 19:09, 25 December 2013 by William J Gibson (talk | contribs) (Created page with "{{WBRQuestion |QuestionAuthor=William J Gibson |ExamType=USMLE Step 1 |MainCategory=Pharmacology |SubCategory=Oncology, Renal |MainCategory=Pharmacology |SubCategory=Oncology,...")
(diff) ← Older revision | Latest revision (diff) | Newer revision → (diff)
Jump to navigation Jump to search
 
Author PageAuthor::William J Gibson
Exam Type ExamType::USMLE Step 1
Main Category MainCategory::Pharmacology
Sub Category SubCategory::Oncology, SubCategory::Renal
Prompt [[Prompt::A 72-year-old man with end-stage Chronic Lymphocytic Leukemia is administered an experimental treatment with tumor-targeting T-cells. Approximately two weeks after receiving the injection, the patient develops a fever of 104 F with shaking and chills. Laboratory studies reveal a thousand-fold increase in IL6 along with the following:

Sodium: 140 mEq/L Potassium: 5.5 mm/L Phosphate: 5.0 mg/dL Calcium: 8.0 mg/dL Uric acid: 16.0 mg/dL (Normal 2.0 – 8.0) Lactic Acid: 5.0 mEq/L (Normal 0.7-2.1)

Which of the following drugs should be avoided in this patient?]]

Answer A AnswerA::Hydrochlorothiazide
Answer A Explanation AnswerAExp::
Answer B AnswerB::Furosemide
Answer B Explanation AnswerBExp::
Answer C AnswerC::Corticosteroids
Answer C Explanation AnswerCExp::
Answer D AnswerD::Allopurinol
Answer D Explanation AnswerDExp::
Answer E AnswerE::Acetazolamide
Answer E Explanation AnswerEExp::
Right Answer RightAnswer::
Explanation [[Explanation::

Educational Objective:
References: ]]

Approved Approved::No
Keyword
Linked Question Linked::
Order in Linked Questions LinkedOrder::